John invest R500, at 12% per annum using simple interest for 4 years. How much money will John have at the end of 4 years?​

Answers

Answer 1

Simple Interest = (Principal x Rate x Time)

where:

Principal = R500 (the amount invested)

Rate = 12% per annum = 0.12 (expressed as a decimal)

Time = 4 years

So, the simple interest earned by John over 4 years is:

Simple Interest = (500 x 0.12 x 4) = R240

To find the total amount of money John will have at the end of 4 years, we need to add the simple interest to the principal:

Total amount = Principal + Simple Interest = 500 + 240 = R740

Therefore, John will have R740 at the end of 4 years.

Answer 2

Answer:

The total amount he has at the end is ₹740.

Step-by-step explanation:

Simple Interest = [tex]\frac{PRT}{100}[/tex]

                          =  [tex]\frac{500 * 12 * 4}{100}[/tex] = ₹240

Total amount he has at the end of 4 years = Principal + Interest
                                                                       = ₹500 + ₹240

                                                                       = ₹740


Related Questions

The length of a rectangular room is 4 feet longer than twice the width. If the room's perimeter is 200 feet, what are the room's dimensions?
OA. Width=64 ft; length=136 ft
OB. Width=32 ft; length=68 ft
OC. Width=37 ft; length = 78 ft
O D. Width=48 ft; length = 52 ft

Answers

Answer:

B

Step-by-step explanation:

Permieter = 2(l + w)

200ft = 2(l + w)

l = 4 + 2w

200ft = 2(4  + 2w + w)

200ft = 2(4 + 3w)

200ft = 8 + 6w

6w = 192ft

w = 32ft

l = 4 + 2(32ft)

= 4 + 64ft

= 68ft

So the answer is B

Hope this helps!

Brainliest and a like is much appreciated!

Please refer to the photo! Thank you

Answers

Answer:

25 flowers and 14 bushes

Step-by-step explanation:

Sarah has $300 budget

flowers go for $6 each

bushes go for $10 each

So lets go through all options and find which combination of flowers and bushes Sarah could buy with her budget.

1. 13 flowers and 23 bushes

13 x 6 = $78

23 x 10 = $230

$78 + $230 = $308 Above $300

2.16 flowers and 21 bushes

16 x 6 = $96

21 x 10 = $210

$96 + $210 = $306 Above $300

3. 8 flowers and 26 bushes

8 x 6 = $48

26 x 10 = $260

$48 + $260 = $308 Above $300

4. 25 flowers and 14 bushes

25 x 6 = $150

14 x 10 = $140

$150 + $140 = $290 Fits the budget so the answer is

25 flowers and 14 bushes


Hope this helps!

Brainliest and a like is much appreciated!

Can someone please hand a help

Answers

The inequality -1/2 ≤ ab ≤ 1/2 implies [tex](ab)^2[/tex] ≤ 1/4, and squaring bοth sides οf the inequality gives us  [tex](ab)^2[/tex] ≤ 1/4 as required.

Tο prοve that -1/2 ≤ ab ≤ 1/2 fοr  [tex]a^2+b^2 = 1[/tex]and a, b ∈ ℝ, we can start by nοting that:

-1 ≤ a ≤ 1 (because [tex]a^2 \le a^2 + b^2 = 1[/tex], sο -1 ≤ a ≤ 1)

-1 ≤ b ≤ 1 (because [tex]b^2 \le a^2 + b^2 = 1[/tex], sο -1 ≤ b ≤ 1)

Multiplying these inequalities, we get:

-1 ≤ ab ≤ 1

Nοw, we need tο shοw that ab cannοt equal ±1. If ab = 1, then we have:

[tex]a^2 + b^2 = 1[/tex]

[tex]a^2 + 2ab + b^2 = 1 + 2ab[/tex]

[tex](a + b)^2 = 1 + 2ab[/tex]

Since a and b are bοth between -1 and 1, a + b is between -2 and 2, sο [tex](a + b)^2[/tex] is between 0 and 4. Therefοre, we have:

1 + 2ab ≤ 4

Simplifying, we get:

ab ≤ 3/2

This cοntradicts the fact that ab = 1, sο ab cannοt equal 1. Similarly, if ab = -1, we get:

[tex](a + b)^2 = 1 - 2ab[/tex]

Since [tex](a + b)^2[/tex] is nοnnegative, we have:

1 - 2ab ≥ 0

Simplifying, we get:

ab ≤ 1/2

This cοntradicts the fact that ab = -1, sο ab cannοt equal -1. Therefοre, we have -1 < ab < 1, which implies -1/2 ≤ ab ≤ 1/2.

Taking the square οf bοth sides οf -1/2 ≤ ab ≤ 1/2, we get:

[tex]1/4 \le a^2b^2 \le 1/4[/tex]

Adding [tex]a^2 + b^2 = 1[/tex]tο bοth sides, we get:

[tex]5/4 \le 1 + a^2b^2 \le 5/4[/tex]

Dividing by 2, we get:

[tex]5/8 \le (1 + a^2b^2)/2 \le 5/8[/tex]

Since [tex](1 + a^2b^2)/2[/tex] is the average οf [tex]a^2[/tex] and [tex]b^2[/tex], we have:

[tex]5/8 \le (a^2 + b^2)/2 \le 5/8[/tex]

Simplifying, we get:

5/8 ≤ 1/2 ≤ 5/8

Therefοre, the inequality -1/2 ≤ ab ≤ 1/2 implies [tex](ab)^2 \le 1/4[/tex], and squaring bοth sides οf the inequality gives us  [tex](ab)^2 \le 1/4[/tex] as required.

Learn more about inequality

https://brainly.com/question/30228778

#SPJ1

Verify that $\triangle ABC\sim\triangle DEF$ . Find the scale factor of $\triangle ABC$ to $\triangle DEF$ .

$\triangle ABC:\ AB\ =\ 10,\ BC\ =\ 16,\ CA\ =\ 20$

$\triangle DEF:\ DE\ =\ 25,\ EF\ =\ 40,\ FD\ =\ 50$

Answers

So the scale factor of triangle ABC to triangle DEF is 2/5.

What is similar triangle?

Similar triangles are two triangles that have the same shape, but not necessarily the same size. This means that their corresponding angles are equal, and their corresponding sides are proportional.

Here,

To verify if the two triangles are similar, we need to check if their corresponding angles are congruent and if their corresponding sides are proportional.

Checking corresponding angles:

∠A corresponds to ∠D

∠B corresponds to ∠E

∠C corresponds to ∠F

Checking corresponding sides:

AB/DE = 10/25 = 2/5

BC/EF = 16/40 = 2/5

CA/FD = 20/50 = 2/5

Since the corresponding angles are congruent and the corresponding sides are proportional, we can conclude that triangle ABC is similar to triangle DEF. To find the scale factor of triangle ABC to triangle DEF, we can take any corresponding side and divide it by the corresponding side of the other triangle. For example:

AB/DE = 2/5

To know more about similar triangle,

https://brainly.com/question/14926756

#SPJ1

Find a quadratic function with vertex (3 -4) and passes through
the point (0,4).

Answers

f(x) = (8/9)(x-3)² - 4 which has a vertex at (3,-4) and passes through the point (0,4).

What is a quadratic equation?

A quadratic equation is a second-degree polynomial equation of the form:

ax² + bx + c = 0

where a, b, and c are constants, and x is the variable. Quadratic equations can have one, two, or zero real solutions, depending on the values of the constants a, b, and c. The solutions can be found using the quadratic formula:

x = (-b ± [tex]\sqrt{b^2 - 4ac}[/tex]) / 2a or by factoring the quadratic expression into two linear factors.

A quadratic function can be expressed in the form:

[tex]$$f(x) = a(x-h)^2 + k$$[/tex]

where (h,k) is the vertex of the parabola.

From the problem, we have the vertex (h,k) = (3,-4). Substituting these values into the equation gives:

[tex]$$f(x) = a(x-3)^2 - 4$$[/tex]

To find the value of a, we use the fact that the function passes through the point (0,4). Substituting x=0 and y=4 into the equation gives:

[tex]$$4 = a(0-3)^2 - 4$$[/tex]

Simplifying and solving for a, we get:

a=8/9

Therefore, the quadratic function is:

f(x) = (8/9)(x-3)² - 4

which has a vertex at (3,-4) and passes through the point (0,4).

To know more about quadratic equations  visit:

brainly.com/question/30098550

#SPJ1

how many rational numbers are there between 0 and 5 explain ur answer in words​

Answers

Answer:

4 or 5

Step-by-step explanation:

because rational numbers also include natural numbers but natural numbers starts from 1 to eternity

while whole numbers start from 0 and is also included in rational numbers so it might be 5

PLS HELP ASAP!
in a circle with radius 7, an angle measuring 5pi/4 radians intercepts an arc. find the length of the arc in simplest form.

Answers

The length of the intercepted arc is (35 - 14√2 ) units.

what is length?

Length is a physical quantity that describes the distance between two points. It is typically measured in units such as meters, centimeters, inches, or feet. In mathematics, length can refer to the size of a geometric object,

In the given question,

We know that the length of an arc of a circle is given by the formula L = r*theta, where r is the radius of the circle and theta is the angle in radians subtended by the arc at the center of the circle.

Here, the radius of the circle is 7 units and the angle subtended by the arc is 5π/4 radians. Therefore, the length of the intercepted arc is:

L = 7*(5π/4) = (35π/4) units.

To express the answer in simplest form, we need to rationalize the denominator. Multiplying both the numerator and denominator by 2√2, we get:

L = (35π/4)(2√2/2√2)

= (35*π*√2)/(8) units.

Finally, simplifying this expression, we get:

L = (35 - 14√2) units.

Therefore, the length of the intercepted arc is (35 - 14√2) units

To know more about length and arc , visit:

https://brainly.com/question/16403495

#SPJ1

The Lozano's chartered a boat over the holiday weekend. The cost for the day was $350 plus the 6.5% sales tax. They also decided to tip their guide 25% of the original cost of their boat rental. What is the total amount they paid?

Answers

Lοzanο's paid a tοtal οf $460.25 fοr the bοat rental, including sales tax and tip.

What are Percentages?

In mathematics, percentages are a way οf expressing a number as a fractiοn οf 100. It is οften used tο cοmpare values, represent prοpοrtiοns, and calculate changes οr discοunts.

The cοst οf the bοat rental fοr the day was $350. The sales tax at a rate οf 6.5% is calculated as:

Sales tax = 6.5% οf $350 = 0.065 × $350 = $22.75

Sο the tοtal cοst οf the bοat rental with sales tax is:

Tοtal cοst = $350 + $22.75 = $372.75

The Lοzanο's alsο decided tο tip their guide 25% οf the οriginal cοst οf the bοat rental, which is:

Tip = 25% οf $350 = 0.25 × $350 = $87.50

Therefοre, the tοtal amοunt they paid is:

Tοtal amοunt = Tοtal cοst + Tip = $372.75 + $87.50 = $460.25Sο the Lοzanο's paid a tοtal οf $460.25 fοr the bοat rental, including sales tax and tip.

To learn more about Percentages from the given link

https://brainly.com/question/24877689

#SPJ1

Answer: $460.25 for the boat rental, including sales tax and tip.

Step-by-step explanation The cost of the boat rental for the day was $350. The sales tax at a rate of 6.5% is calculated as:

Sales tax = 6.5% of $350 = 0.065 × $350 = $22.75

So the total cost of the boat rental with sales tax is:

Total cost = $350 + $22.75 = $372.75

The Lozano's also decided to tip their guide 25% of the original cost of the boat rental, which is:

Tip = 25% of $350 = 0.25 × $350 = $87.50

Therefore, the total amount they paid is:

Total amount = Total cost + Tip = $372.75 + $87.50 = $460.25

So the Lozano's paid a total of $460.25 for the boat rental, including sales tax and tip.

Find the gross income for selling 348 bushels of apples at 16. 50 per bushel

Answers

The gross income for selling 348 bushels of apples at $16.50 per bushel is $5,732.

The formula to calculate gross income from the sale of a certain number of bushels of apples at a certain price per bushel is Gross Income = Number of Bushels x Price per Bushel. In this case, the calculation of the gross income for selling 348 bushels of apples at $16.50 per bushel would be Gross Income = 348 Bushels x $16.50/Bushel = $5,732.

To calculate the gross income, first we must multiply the number of bushels of apples (348) by the price per bushel ($16.50). When multiplied, the numbers result in $5,732. This means that, when selling 348 bushels of apples at $16.50 per bushel, the gross income would be $5,732.

Learn more about gross income here:

https://brainly.com/question/15821021

#SPJ4

The figure below is a net for a triangular pyramid. 6.93 m 8 m If all the triangles are equilateral, what is the surface area of the pyramid, in square meters?​

Answers

Answer:

Step-by-step explanation:

what is the sum of 22+54x= -20+60x

Answers

Answer:

x=7

Step-by-step explanation:

22+54x=-20+60x

42+54x=60x

42=6x

x=7

The time to failure for a power supply unit used in a particular brand of personal computer (PC) is thought to be exponentially distributed with a mean of 4,000 hours as per the contract between the vendor and the PC maker. The PC manufacturer has just had a warranty return from a customer who had the power supply fail after 2,100 hours of use.
a. What is the probability that the power supply would fail at 2,100 hours or less? Based on this probability, do you feel the PC maker has a right to require that the power supply maker refund the money on this unit?
b. Assuming that the PC maker has sold 100,000 computers with this power supply, approximately how many should be returned due to failure at 2,100 hours or less?

Answers

The probability that the power supply would fail at 2,100 hours or less is approximately 33.6%.

What is the probability that the power supply would fail at 2,100 hours or less? Based on the provided information, the time to failure for a power supply unit used in a particular brand of personal computer (PC) is assumed to be exponentially distributed with a mean of 4,000 hours. Therefore,λ = 1/4000 = 0.00025. Now we can use the exponential distribution function to calculate the probability that the power supply would fail at 2,100 hours or less. Therefore, P(X ≤ 2100) = F(2100) = 1 - e^(-λx) = 1 - e^(-0.00025 × 2100) = 0.336 = 33.6%.Therefore, the probability that the power supply would fail at 2,100 hours or less is approximately 33.6%.

Based on this probability, it can be concluded that the PC maker has a right to require that the power supply maker refund the money on this unit.b. Assuming that the PC maker has sold 100,000 computers with this power supply, approximately how many should be returned due to failure at 2,100 hours or less?From part a, we know that the probability of failure at 2,100 hours or less is 0.336. Therefore, the number of PCs that are likely to fail at 2,100 hours or less can be calculated as follows:Number of PCs that will fail at 2,100 hours or less = 0.336 × 100,000 = 33,600 Hence, approximately 33,600 PCs should be returned due to failure at 2,100 hours or less.

Learn more about Probability

brainly.com/question/11234923

#SPJ4

someone pls help (13 points )

Answers

Answer:

y = 1x-5

Step-by-step explanation:

See attached worksheet.

We'll look for a line with the form y=mx+b, where m is the slope and y is the y-intercept.

Pick any two points on the line, but pick ones that are clearly on known lines so that the points are more accurate.  Pick one at the x=o point, if it can be read clearly.  The vaue of y at x=0 is the y-intercept.

Follow the steps in the attachement to find the equation of the line, which is

  y=1x-5

Carlos put $400,000 into a savings account for his daughter when she was born. The account earns 3. 5% interest compounded annually. He will give his daughter access to the account when she graduates high school. How much will his daughter receive upon graduation?

Answers

Carlos' daughter will receive $847,438.00 from the savings account.

The formula used to calculate the amount of money earned through compound interest is [tex]A = P(1 + r/n)^nt[/tex], where A is the amount of money earned, P is the principal amount, r is the rate of interest per annum, n is the number of times the interest is compounded per year, and t is the number of years.

In Carlos' case, P = 400,000, r = 3.5%, n = 1, and t = 18 (the number of years between when his daughter was born and when she graduates high school). Plugging these values into the formula above, we get:

[tex]A = 400,000(1 + 0.035/1)^(1*18)A = 400,000(1.035)^18\\A = 400,000(2.11859)\\A = $847,438.00[/tex]

Therefore, upon graduation, Carlos' daughter will receive $847,438.00 from the savings account.

Learn more about compound interest here:

https://brainly.com/question/28792777

#SPJ4

Using the bag of marbles below, find the probability of pulling a red marble two times in a row. After pulling the first red marble, you do NOT replace it in the bag before pulling the second one. Each marble has an equally likely chance of being pulled.

A. 10/21
B. 5/7
C. 25/51
D. 4/63​

Answers

Answer:

A. 10/21

Step-by-step explanation:

There is a total of 7 marbles, 5 of which are red ---> 5/7

When you pull out a red marble and do not replace it in the bag before pulling the second one, a marble (red) is gone hence ----> 4/6

Multiply the two events to find the probability:

5/7 x 4/6 = 10/21

PLEASE HELP ME ! I'm struggling so much with my hmwk this week and needs help!
2.5 m³ of granite has a mass of 6725 kg.
a) Calculate the density of granite in kg/m
b) Find the mass of 1.3 m³ of granite in kg.​

Answers

Answer:

Below

Step-by-step explanation:

a) Density =  mass/ volume

  you are given  mass ( 6725kg) and volume (2.5 m^3) and you want

              kg / m^3    this would be   6725 kg / 2.5 m^3 = 2690 kg/m^3

b) given density = 2690 kg/m^3    multiply by m^3 to get  kg

                               2690 kg / m^3   *  1.3 m^3  = 3497 kg

This composite figure is created by placing a sector of a circle on a triangle. What is the area of this composite figure? Use 3.14 for π. Round to the nearest hundredth. Show your work.

Answers

The area of the composite figure is 96. 522 cm²

How to determine the area

First, we need to know that the formula for the area of a triangle is expressed as;

Area = 1/2 × base × height

Now, substitute the values, we have;

Area = 1/2 × 6 × 8

Multiply the values

Area = 1/ 2 × 48

Divide the values

Area = 25 cm²

The area of the sector is represented as;

Area = θ/360 πr²

substitute the values

Area = 82/360 × 3.14 × 10²

Area = 71. 522 cm²

The total area of the figure = 25 + 71. 522 = 96. 522 cm²

Learn about area at: https://brainly.com/question/22972014

#SPJ1

I need help with this please.
It’s question one and two

Answers

In step 4 and 5 he used prime factorization of 5 under cube root.

This was done in order to eliminate the cube root.

What is prime factorization method?

Prime factorization is a process of finding the prime factors of a given number. A prime factor is a factor of a number that is a prime number. The prime factorization of a number is the list of its prime factors, together with their multiplicities.

To find the prime factorization of a number, divide it by its smallest prime factor and write down the result as the product of the prime factor and the quotient. Continue this process with the quotient until quotient of 1 is obtained.  

Find out more on prime factorization here: https://brainly.com/question/18187355

#SPJ1

The equation x + (75.3 - x) = 75.3 represents the sum of the measures
of two angles. How many possible combinations of angle measures satisfy
the equation?

Answers

Overall, there are infinitely many possible combinations of angle measures that satisfy the equation x + (75.3 - x) = 75.3.

How many possible combinations of angle measures satisfy the equation?

It is true that 75.3 = 75.3 when the equation x + (75.3 - x) = 75.3 is reduced to its simplest form. This means that any value of x that falls within the range of possible angle measurements, which is 0 to 75.3 degrees, will satisfy the equation.

As a result, the number of possible combinations of angle measurements that fulfil the equation is unlimited. The particular values of each angle can vary, but each combination will consist of two angles whose measures total up to 75.3 degrees.

For instance, if the sum of the two angles is 75.3 degrees, one conceivable angle combination is 30 degrees and 45.3 degrees. The combination of 60 degrees and 15.3 degrees is another conceivable one.

Learn more about linear equation here:

brainly.com/question/11897796

#SPJ9

Oint

MRS. JAMISON'S CLASS

MR. ZIMMERMAN'S CLASS

0

+ +

1 2 3 4 5 6

NUMBER OF TVs PER HOUSEHOLD

7

0 1 2 3

5 6

NUMBER OF TVs PER HOUSEHOLD

What is the median number of TVs in Mrs. Jamison's class?

Answers

The median number of TVs in Mrs. Jamison's class is 2, (i.e. the middle number).

To find the median of the given data of the number of TVs in Mrs. Jamison's class:

First we have to list all the values from least to greatest.

Then, find the middle number (in case if there is one middle number) then that will be the median of the data.

And if there is two middle number, we have to find the average of the two middle numbers and the result will be our answer i.e. median.

So, The data is 0,0,0,1,1,1,1,1,1,2,2,2,2,2,2,2,3,3,3,3,3,4,4,5,6

The middle number from the data is 2

                        Therefore, the median is 2.

To know more about Median:

brainly.com/question/14532771

#SPJ4

Suppose the following information is collocted from $186,928 is what type of variable? Robert on his application for A. Numerical, discrote, interval a loan. a. B. Numerical, continucus, Annual income payments: interval $186,928 b. Number of credit cards: 4 C. Categorical, nominal c. Ever convicted of a felony: D. Numerical, discrete, ratio No E. Numerical, continuous, ratio d. Own a second car. No Classify

Answers

The variable "Annual income payments: $186,928" is of the type "Numerical, continuous, ratio."

The given information that Robert filled on his application for a loan is as follows:Annual income payments: $186,928Number of credit cards: 4Ever convicted of a felony: NoOwn a second car: NoThe types of variables are:Numerical: A numerical variable is a measurable quantity, and they are of two types: discrete and continuous.Discrete variables: They are quantitative variables that can be counted and are usually whole numbers. In this case, there is no discrete numerical variable, so it is not a valid option.Continuous variables: They are quantitative variables that can be measured and are usually decimal. In this case, the only continuous numerical variable is the annual income payments which amounts to $186,928.Categorical: A categorical variable is a variable that is defined by how the categories can be defined. It does not have a numerical meaning, unlike a numerical variable. It is further divided into two types: nominal and ordinal.Nominal variables: They are variables that have no order or ranking. In this case, the ever convicted of a felony is a nominal variable.Ordinal variables: They are variables that have an order or ranking. In this case, there is no ordinal categorical variable, so it is not a valid option.Therefore, the variable "Annual income payments: $186,928" is of the type "Numerical, continuous, ratio." The variable "Ever convicted of a felony" is of the type "Categorical, nominal."

Learn more about Annual income payments

brainly.com/question/11281245

#SPJ11

Write a vertical motion model in the form h(t)=-16t^2+v0^2+h0 for each situation presented. For each situation, determine how long, in seconds, it takes the thrown object to reach maximum height.

Initial velocity: 120 ft/s; initial height: 50 ft

Answers

Answer:

-16t^2 + 120t + 50

Step-by-step explanation:

Using the given values, we can write the model for this situation as:

h(t) = -16t^2 + 120t + 50

To determine how long it takes the thrown object to reach maximum height, we need to find the time at which the object reaches its maximum height. The maximum height occurs at the vertex of the parabolic path, which is given by:

t = -b / 2a

where a = -16, b = 120.

Substituting these values, we get:

t = -120 / 2(-16) = 3.75 seconds

Therefore, it takes 3.75 seconds for the thrown object to reach maximum height.

Please help!! Will mark branliest

Answers

The real and imaginary values of w is 86  and - 0.92i respectively.

What is the real and imaginary values of w

To find w = √(3 - 4i), we can use the following steps:

Step 1: Find the modulus and argument of z = 3 - 4i

The modulus of z is

|z| = √(3² + (-4)²)

= √(9 + 16) = √25

= 5.

The argument of z is arg(z) = arctan(-4/3) ≈ -0.93 radians (or about -53.13 degrees).

Step 2: Find the principal square root of the modulus of z, which is

√|z| = √5.

Step 3: Find the argument of w, which is half of the argument of z, i.e., arg(w) = arg(z)/2

= -0.93/2

≈ -0.465 radians (or about -26.57 degrees).

Step 4: Express w in terms of its real and imaginary parts, using the formula:

w = √|z| * exp(i*arg(w)).

Substituting the values we found above, we get:

w = √5 x exp(i(-0.465))

= √5 x (cos(-0.465) + i*sin(-0.465))

≈ 1.86 - 0.92i

Learn more about real and imaginary values here: https://brainly.com/question/5564133

#SPJ1

F(x)= 1/2sin(0/3-90)+1

Answers

The value of f(0) for the given function is 1÷2.

What is Function?

In mathematics, a function is a relation between two sets of values, where each value in the first set (called the domain) is associated with exactly one value in the second set (called the range). A function can be represented by a formula or an equation, which specifies how the input values are transformed into output values.

To find the value of f(0) for the given function f(x) = 1/2sin(x÷3-90)+1, we need to substitute 0 for x in the expression for f(x) and simplify:

f(0) = 1÷2sin(x÷3-90)+1

f(0) = 1÷2sin(-90)+1

f(0) = 1÷2(-1)+1 [since sin(-90) = -1]

f(0) = -1÷2 + 1

f(0) = 1÷2

Therefore, the value of f(0) for the given function is 1÷2.

To learn more about Function from given link.

brainly.com/question/29120892

#SPJ1

Complete question:

Find the value of F(0) when F(x)= 1/2sin(0/3-90)+1.

A right triangle has side lengths A B and C use these links to find cosx tanx and sinx

Answers

In a right triangle, the side opposite the right angle is called the hypotenuse and is labeled as `C`. The other two sides are called the legs and are labeled as `A` and `B`. The angle opposite side `A` is labeled as `x`.

To find `cos(x)`, `sin(x)`, and `tan(x)` in terms of `A`, `B`, and `C`, you can use the following trigonometric formulas:

- `cos(x) = A/C`

- `sin(x) = B/C`

- `tan(x) = sin(x) / cos(x) = B/A`

So if you know the lengths of the sides `A`, `B`, and `C`, you can use these formulas to find `cos(x)`, `sin(x)`, and `tan(x)`.

For example, if `A = 3`, `B = 4`, and `C = 5` (which is a Pythagorean triple), then the angle `x` opposite `A` is the angle whose cosine, sine, and tangent we wish to find. Using the formulas above, we have:

- `cos(x) = A/C = 3/5`

- `sin(x) = B/C = 4/5`

- `tan(x) = sin(x) / cos(x) = (4/5) / (3/5) = 4/3`

Therefore, in this case, `cos(x) = 3/5`, `sin(x) = 4/5`, and `tan(x) = 4/3`.

Write the equation of the line that is PARALLEL to y = -4x + 5 and passes through the point (-2,-1)

Answers

Answer: Two parallel lines have the same slope. Therefore, we can use the slope of the given line y = -4x + 5 to find the slope of the line that is parallel to it.

The slope of y = -4x + 5 is -4. Therefore, the slope of any line parallel to it will also be -4.

Now we have the slope of the line and a point that it passes through. We can use point-slope form to write the equation of the line:

y - y1 = m(x - x1)

where m is the slope and (x1, y1) is the point on the line.

Plugging in the values we know, we get:

y - (-1) = -4(x - (-2))

y + 1 = -4(x + 2)

y + 1 = -4x - 8

y = -4x - 9

Therefore, the equation of the line that is parallel to y = -4x + 5 and passes through the point (-2,-1) is y = -4x - 9.

Step-by-step explanation:

9. Sydney invests $100 every month into an account that pays 5% annual interest, compounded monthly. Benny invests $80 every month into an account that pays 8% annual interest rate, com- pounded monthly. a. Determine the amount in Sydney's account after 10 years. b. Determine the amount in Benny's account after 10 years. c. Who had more money in the account after 10 years? d. Determine the amount in Sydney's account after 20 years. e. Determine the amount in Benny's account after 20 years. f. Who had more money in the account after 20 years?​

Answers

Answer: a. To determine the amount in Sydney's account after 10 years, we can use the formula for compound interest:

FV = PMT × (((1 + r/n)^(n*t) - 1) / (r/n))

where FV is the future value, PMT is the monthly payment, r is the annual interest rate, n is the number of times compounded per year, and t is the number of years.

Plugging in the values for Sydney's account, we get:

FV = 100 × (((1 + 0.05/12)^(12*10) - 1) / (0.05/12))

FV = $16,184.46

Therefore, the amount in Sydney's account after 10 years is $16,184.46.

b. To determine the amount in Benny's account after 10 years, we can use the same formula:

FV = PMT × (((1 + r/n)^(n*t) - 1) / (r/n))

Plugging in the values for Benny's account, we get:

FV = 80 × (((1 + 0.08/12)^(12*10) - 1) / (0.08/12))

FV = $15,710.21

Therefore, the amount in Benny's account after 10 years is $15,710.21.

c. Sydney had more money in the account after 10 years, since $16,184.46 > $15,710.21.

d. To determine the amount in Sydney's account after 20 years, we can use the same formula:

FV = PMT × (((1 + r/n)^(n*t) - 1) / (r/n))

Plugging in the values for Sydney's account, we get:

FV = 100 × (((1 + 0.05/12)^(12*20) - 1) / (0.05/12))

FV = $45,074.89

Therefore, the amount in Sydney's account after 20 years is $45,074.89.

e. To determine the amount in Benny's account after 20 years, we can use the same formula:

FV = PMT × (((1 + r/n)^(n*t) - 1) / (r/n))

Plugging in the values for Benny's account, we get:

FV = 80 × (((1 + 0.08/12)^(12*20) - 1) / (0.08/12))

FV = $42,598.05

Therefore, the amount in Benny's account after 20 years is $42,598.05.

f. Sydney had more money in the account after 20 years, since $45,074.89 > $42,598.05.

Step-by-step explanation:

Complete the identity. cos (2pi - x) =?

Please I need help this test is due! :/

I need help with number 5

Answers

the identity is:

cos(2π - x) = cos(x)

Using the angle sum identity for cosine, we have:

cos(2π - x) = cos(2π)cos(x) + sin(2π)sin(x)

Since cos(2π) = 1 and sin(2π) = 0, this simplifies to

cos(2π - x) = cos(x)

what is cosine?

The ratio between the adjacent side and the hypotenuse is known as the cosine function (or cos function) in triangles. One of the three fundamental trigonometric functions, cosine is the complement of sine (co+sine) and one of the three main trigonometric functions.

The ratio of the neighboring side's length to the longest side, or hypotenuse, in a right triangle is known as the cosine. Let's say that the hypotenuse of a triangle ABC is written as AB, and the angle between the hypotenuse and base is written as.

It's interesting to observe that the cosv value varies depending on the quadrant. As observed , cos 0°, 30°, etc. have positive values while cos 120°, 150°, and 180° have negative values. Cos will have a good value in the first and fourth quadrants.

from the question:

Using the angle sum identity for cosine, we have:

cos(2π - x) = cos(2π)cos(x) + sin(2π)sin(x)

Since cos(2π) = 1 and sin(2π) = 0, this simplifies to:

cos(2π - x) = cos(x)

Therefore, the identity is:

cos(2π - x) = cos(x)

to learn more about cosine visit

https://brainly.com/question/29114352

#SPJ1

Vincent deposited $1, 650 in a savings account that earns 3.5%interest compounded daily for 90 days. Find the maturity
value after the money has been in Vincent's account for 90 days. Use the table below.
Interest by Quarter for 3.5% Compounded Daily
Assuming 90-day Quarters
Number of Quarters
1
2
3
4
$1,698.71
$1,632.46
$1,664.30
$1,678.92
Value of (1 + i)"
1.008667067
1.017409251
1.026227205
1.035121585

Answers

To solve this problem, we will use the formula for compound interest:

A = P(1 + r/n)^(nt)

Where:
A = Maturity value (what we are trying to find)
P = Principal (initial deposit) = $1,650
r = Annual interest rate = 3.5%
n = Number of times the interest is compounded in a year = 365 (since interest is compounded daily)
t = Time (in years) = 90/365 = 0.246575

Plugging in the values, we get:

A = $1,650(1 + 0.035/365)^(365*0.246575)
A = $1,698.71 (rounded to the nearest cent)

Therefore, the maturity value after 90 days is $1,698.71.

Janet is frosting the top layer of 4 rectangular birthday cakes. If each cake measures 13 inches by 8 inches, how many square inches of cake will be covered by frosting?

Answers

Janet will cover 416 square inches of rectangular cake with frosting.

What exactly is a rectangle?

A rectangle is a geometric shape that has four sides and four right angles (90 degrees). It is a type of quadrilateral, which means a four-sided polygon. The opposite sides of a rectangle are parallel and equal in length, and the adjacent sides are perpendicular to each other.

The area of a rectangle can be calculated by multiplying its length and width (or base and height) together, while its perimeter is the sum of the lengths of all four sides.

Now,

Since Janet is frosting the top layer of 4 rectangular birthday cakes, we need to calculate the total area of the top layer of all 4 cakes combined.

Each cake measures 13 inches by 8 inches, so the area of one cake is:

13 inches x 8 inches = 104 square inches

The top layer of one cake will have the same dimensions, so the area of the top layer of one cake is also 104 square inches.

To find the total area of the top layer of all 4 cakes combined, we can multiply the area of one cake by 4:

Total area = 104 square inches/cake x 4 cakes = 416 square inches

Therefore, Janet will cover 416 square inches of cake with frosting.

To know more about rectangles visit the link

brainly.com/question/29123947

#SPJ1

Other Questions
What is Swifts attitude toward England? Cite one or more specific passages in the text where he offers his thoughts, either directly or indirectly, about English rule. Find the area of the triangle whose vertices are (2,3),(3,2) and (1,8) by using determinant method. CivicsThe Qin Dynasty revolutionized how China would govern itself through complex bureaucracy.Explain how this system worked and describe some of its major accomplishments. Do you think that modern racism led to American Imperialism or that American Imperialism had to use modern racism to justify its actions? Which of these is not true about a polynomial regression model?Group of answer choicesa. used when the linear regression is not able to capture the data pointsb. has multiple predictorsc. requires a linear relationship between the predictor and the dependent variable Translate this sentence into a inequality. The product of x and 4 is greater than or equal to 18 Theoretical propobility Which detail from Too Green for Basketball best supports the implied theme?"A flash of neon green shot across the court."Maybe not, but its not right to leave someone out."In fact, they could tell that any team with Frankie on it would win."Good game, Ashley finally said. Same time tomorrow, everybody? what is the minimum weather condition required for airplanes operating under special vfr in class d airspace? The long-run price elasticity of supply of automobiles is _____the short-run price elasticity of supply of automobiles. 1. ALUMMUNI PLC. Produces three models of tractors: Metakeb, Mewesson, Metekem Each unit of Metakeb, Mewesson and Metekem requires the following amounts of time in minumtes in each of the indicated departments.Machining dep'tInspection dep't(in minutes)(in minutes)(in minutes)Metakeb12002400600Mewesson180012003000Metekem3000Assembly dep't24001200Suppose the total time available per month in machining, assembly and inspection departments are 1050, 1160 and 830 hours respectively.Required:Determine the number of units of each product to be produced in a month to use up all the available resources (use Gaussaian method) Lara deposits $49,000 to be compounded monthly at 15.3% for 2 years.Which equation will she use to determine how much money she'll have after 2 years? iAfter 2 years she'll have ___ Describe 2 specific ethnic groups of people that immigrated to the US before 1870 and explain the PUSH FACTOR for each group A contract requires lease payments of $800 at the beginning of every month for 4 years.a. What is the present value of the contract if the lease rate is 4.50% compounded annually?b. What is the present value of the contract if the lease rate is 4.50% compounded monthly? HELP ME PLEASE! Rockville and Marion are 7cm apart on a map that has a scale of 1 in: 3ft, then how tall is the real house? How would you describe the importance of plasticity to the nervous system? what is the npv of a 6-year project that costs $100,000, has annual revenues of $50,000, and annual costs of $15,000? assume the investment is an asset that will be depreciated straight-line over 6 years to a salvage value of $0, the corporate tax rate is 21% and the discount rate is 14%. round you answer to the nearest whole dollar amount (zero decimal places). 2.1 Define the term "bursary" and state ONE factor that is commonly taken into considered when awarding a bursary to an applicant (1+1)(2) ? QUESTION Divide. Write your answer in simplest form. 3-:(7)/(3) OO EXPLANATION Where was the trap laid?